03 octubre 2014

Fuerza entre dos cargas en movimiento

1 Enunciado

Dos cargas puntuales iguales q1 y q2 se mueven con la misma celeridad v de forma que en un instante se encuentran situadas en \mathbf{r}_1=\mathbf{0} y \mathbf{r}_2 = a\mathbf{u}_{x}, respectivamente.
Si las dos cargas se mueven con velocidades pequeñas \mathbf{v} = v\mathbf{u}_{z}, calcule el valor aproximado de la fuerza eléctrica y de la fuerza magnética que ejerce cada carga sobre la otra. ¿Cuál es la proporción entre estas dos fuerzas?
¿Cómo cambian estas fuerzas si se cambia el signo de una de las cargas, el sentido de una de las velocidades, o ambas cosas a la vez?
Calcule el valor de estas fuerzas si \mathbf{v}_1=v\mathbf{u}_{z}, \mathbf{v}=v\mathbf{u}_{x}. ¿Se verifica la tercera ley de Newton?

2 Fuerza magnética

2.1 Directamente

Podemos hallar la fuerza magnética entre las dos cargas empleando la Ley de Biot y Savart para cargas puntuales. La fuerza que la carga 1 produce sobre la 2 es
\mathbf{F}_{\mathrm{m}21} = \frac{\mu_0}{4\pi}\,\frac{q_1q_2\mathbf{v}_2\times(\mathbf{v}_1\times(\mathbf{r}_2-\mathbf{r}_1))}{|\mathbf{r}_2-\mathbf{r}_1|^3}
donde en nuestro caso tenemos, tomando como Z el que pasa por la carga 1 y paralelo a su velocidad, y como eje X el que va de la carga 1 a la 2,
\mathbf{r}_2-\mathbf{r}_1=a\mathbf{u}_x        |\mathbf{r}_2-\mathbf{r}_1|=a        \mathbf{v}_1=\mathbf{v}_2=v\mathbf{u}_z\,
Sustituyendo nos queda
\mathbf{F}_{\mathrm{m}21}= \frac{\mu_0q_1q_2v^2}{4\pi a^2}\,\mathbf{u}_z\times(\mathbf{u}_z\times\mathbf{u}_x) = -\frac{\mu_0q_1q_2v^2}{4\pi a^2}\,\mathbf{u}_x
donde hemos usado los productos entre los vectores de la base. Esta fuerza sobre la carga 2 va en la dirección de la 1, es decir, es atractiva.
Para hallar la fuerza que la carga 2 produce sobre la 1, lo único que cambia es que el vector de posición relativo es ahora -a\mathbf{u}_x, por lo que resulta una fuerza
\mathbf{F}_{\mathrm{m}12}=\frac{\mu_0q_1q_2v^2}{4\pi a^2}\,\mathbf{u}_x = -\mathbf{F}_{21}
Esta fuerza es también atractiva, cumplie´ndose además la tercera ley de Newton.
Generalizando este resultado tenemos que las fuerzas entre corrientes paralelas y en el mismo sentido, son atractivas.

2.2 Empleando el campo magnético

Podemos escribir el resultado de una forma más clara, desglosando el cálculo con ayuda del campo magnético como intermediario. Consideramos que la carga 1 produce un campo magnético, que es percibido por la carga 2.
El campo magnético de la carga 1 en todo el espacio es
\mathbf{B}_1(\mathbf{r})=\frac{\mu_0}{4\pi}\,\frac{q_1\mathbf{v}_1\times(\mathbf{r}-\mathbf{r}_1)}{|\mathbf{r}-\mathbf{r}_1|^3}
y en la posición de la 2ª es
\mathbf{B}_1\left(\mathbf{r}_2\right) = \frac{\mu _0q_1v_1}{4\pi a^2}\mathbf{u}_y
La fuerza magnética sobre la carga 2, de acuerdo con la ley de Lorentz
\mathbf{F}_{\mathrm{m}2}=q_2\mathbf{v}_2\times\mathbf{B}(\mathbf{r}_2)
que, sustituyendo el campo anterior nos da lo que ya conocemos
\mathbf{F}_{\mathrm{m}21} = q_2\mathbf{v}_2 \times \mathbf{B}_1\left(\mathbf{r}_2\right) =  - \frac{\mu _0q_1q_2v^2}{4\pi a^2}\mathbf{u}_x

3 Comparación con la fuerza eléctrica

Podemos comparar la magnitud de esta fuerza con la fuerza eléctrica entre ambas cargas. Si la velocidad de las cargas es pequeña, podemos aproximar la fuerza mediante la ley de Coulomb:
\mathbf{F}_{\mathrm{e}21} = \frac{q_1q_2}{4\pi\varepsilon_0a^2}\mathbf{u}_x
La proporción entre ambas fuerzas es
\frac{F_\mathrm{m}}{F_\mathrm{e}}=\mu_0\varepsilon_0 v^2=\frac{v^2}{c^2}
donde c = 3\times 10^{8}\mathrm{m}/\mathrm{s} es la velocidad de la luz. Por ello, normalmente
F_\mathrm{m}\ll F_\mathrm{e}
Las fuerzas magnéticas sólo son apreciables porque la materia es neutra. En la interacción entre dos hilos conductores de corriente, las fuerzas eléctricas entre iones y electrones se cancelan (y F_\mathrm{e}\simeq 0). Para la fuerza magnética sólo cuentan los electrones móviles, y F_\mathrm{m}\neq 0.

4 Cambio de signo

Si cambiamos el signo a una de las cargas la fuerza magnética cambia de signo, pasando a ser repulsiva. Lo mismo ocurre si cambiamos el sentido a una de las velocidades. Si cambiamos el signo de una carga y de una velocidad, la fuerza vuelve a ser atractiva.
Teniendo en cuenta que la densidad de corriente va como el producto de la carga por la velocidad, este resultado predice que corrientes paralelas (↑↑) se atraen, mientras que las antiparalelas (↑↓) se repelen.

5 Velocidades perpendiculares

5.1 Directamente

Aplicando de nuevo la expresión de la fuerza entre dos cargas en movimiento
\mathbf{F}_{\mathrm{m}21} = \frac{\mu_0}{4\pi}\,\frac{q_1q_2\mathbf{v}_2\times(\mathbf{v}_1\times(\mathbf{r}_2-\mathbf{r}_1))}{|\mathbf{r}_2-\mathbf{r}_1|^3}
donde tenemos ahora, al igual que antes
\mathbf{r}_2-\mathbf{r}_1=a\mathbf{u}_x        |\mathbf{r}_2-\mathbf{r}_1|=a        \mathbf{v}_1=v\mathbf{u}_z\,
siendo el único cambio
\mathbf{v}_2=v\mathbf{u}_x\,
que al sustituir nos da
\mathbf{F}_{\mathrm{m}21}= \frac{\mu_0q_1q_2v^2}{4\pi a^2}\,\mathbf{u}_x\times(\mathbf{u}_z\times\mathbf{u}_x) = \frac{\mu_0q_1q_2v^2}{4\pi a^2}\,\mathbf{u}_z
Si hallamos ahora la fuerza que la 2 produce sobre la 1
\mathbf{F}_{\mathrm{m}12}= \frac{\mu_0q_1q_2v^2}{4\pi a^2}\,\mathbf{u}_z\times(\mathbf{u}_x\times(-\mathbf{u}_x)) =\mathbf{0}
Obtenemos que la fuerza que la 1 produce sobre la 2 es no nula, pero la que la 2 produce sobre la 1 sí lo es, es decir, que no se cumple la 3ª ley de Newton.
\mathbf{F}_{\mathrm{m}21}\neq \mathbf{F}_{\mathrm{m}12}

5.2 Empleando el campo magnético

Lógicamente, obtenemos el mismo resultado empleando el campo magnético como intermediario, pero con una interpretación diferente. Para la fuerza que la 1 produce sobre la 2,
\mathbf{F}_{\mathrm{m}21}=q_2\mathbf{v}_2\times\mathbf{B}_1(\mathbf{r}_2)
el campo magnético de la 1 en la posición de la 2 es el mismo que en el caso de las velocidades paralelas. Simplemente cambiamos \mathbf{v}_2 por una perpendicular, resultando una fuerza perpendicular a la que obtuvimos en el primer apartado.
\mathbf{F}_{\mathrm{m}21}=q_2v\mathbf{u}_x\times\left(\frac{\mu_0q_1v}{4\pi a^2}\mathbf{u}_y\right)=\frac{\mu_0q_1q_2v^2\mathbf{u}_z}{4\pi a^2}
Para la fuerza que la 2 produce sobre la 1, consideramos el campo producido en todo el espacio por una carga en movimiento
\mathbf{B}=\frac{\mu_0qv\rho\mathbf{u}_\varphi}{4\pi(\rho^2+z^2)^{3/2}}
que nos dice que el campo magnético de una carga es nulo en todos los puntos del eje del movimiento (salvo en la propia carga). Pero la carga q1 se encuentra situada justamente sobre este eje, por lo que
\mathbf{F}_{\mathrm{m}12}=q_1\mathbf{v}_1\times\mathbf{B}_2(\mathbf{r}_1)=q_1\mathbf{v}_1\times\mathbf{0}=\mathbf{0}
En esta descripción de la interacción, la anulación de la fuerza no se debe a que las velocidades sean paralelas o perpendiculares, sino a que el campo de una de las cargas se anula en la posición de la otra, no ocurriendo lo recíproco.

6 Observaciones

6.1 Violación de la 3ª ley

El que las fuerzas magnéticas no sean iguales y opuestas implica que no se cumple la 3ª ley de Newton; puesto que esta ley es esencial para la conservación de la cantidad de movimiento, para salvar esta ley es necesario introducir una cantidad de movimiento electromagnética asociada al propio campo. De esta forma, el campo se lleva toda la cantidad de movimiento que pierde el sistema mecánico (o aporta la que gana).

6.2 ¿Velocidades absolutas o relativas?

En la fuerza magnética entre cargas puntuales es llamativo que aunque en la fuerza magnética sí aparece la posición relativa de las dos cargas, \mathbf{r}_2-\mathbf{r}_1, no aparece la velocidad relativa \mathbf{v}_2-\mathbf{v}_1, sino las velocidades absolutas \mathbf{v}_1 y \mathbf{v}_2. La pregunta inmediata es ¿respecto a qué se mueven estas cargas? ¿Existe un sistema de referencia absoluto? Una de las enseñanzas de la teoría de la relatividad es que no existen los sistemas de referencia absolutos. Si cambiamos de sistema de referencia absoluto resulta que la fuerza magnética valdrá más o menos o incluso anularse. Este comportamiento claramente no relativista de la fuerza magnética no puede ser correcto.
Sin profundizar en la teoría de la relatividad, podemos salvar este problema, observando que
  • Lo que tiene significado es la fuerza total, suma de la fuerza magnética y de la eléctrica
\mathbf{F}_{21}=\mathbf{F}_{\mathrm{e}21}+\mathbf{F}_{\mathrm{m}21}
  • La fuerza eléctrica también depende de la velocidad de las cargas (la ley de Coulomb es solo una aproximación), de forma que al cambiar de sistema de referencia no solo cambia la fuerza magnética, sino también la eléctrica (en una cantidad similar), de manera que la resultante sí se transforma como debe hacerlo una fuerza relativista.

Tomado de: http://laplace.us.es/wiki/index.php/Fuerza_entre_dos_cargas_en_movimiento

No hay comentarios:

Publicar un comentario